Difference between revisions of "2016 AMC 8 Problems/Problem 5"

m (Solution)
m (Solution)
Line 12: Line 12:
 
==Solution==
 
==Solution==
 
From the second bullet, we know that the second digit must be 3. Because there is a remainder of 1 when it is divided by three, the multiple of 9 must end in a 2. We now look for this one:  
 
From the second bullet, we know that the second digit must be 3. Because there is a remainder of 1 when it is divided by three, the multiple of 9 must end in a 2. We now look for this one:  
 +
 
<math>9(1)=9\\
 
<math>9(1)=9\\
 
9(2)=18\\
 
9(2)=18\\

Revision as of 10:13, 23 November 2016

The number $N$ is a two-digit number.

• When $N$ is divided by $9$, the remainder is $1$.

• When $N$ is divided by $10$, the remainder is $3$.

What is the remainder when $N$ is divided by $11$?


$\textbf{(A) }0\qquad\textbf{(B) }2\qquad\textbf{(C) }4\qquad\textbf{(D) }5\qquad \textbf{(E) }7$

Solution

From the second bullet, we know that the second digit must be 3. Because there is a remainder of 1 when it is divided by three, the multiple of 9 must end in a 2. We now look for this one:

$9(1)=9\\ 9(2)=18\\ 9(3)=27\\ 9(4)=36\\ 9(5)=45\\ 9(6)=54\\ 9(7)=63\\ 9(8)=72$

The number $72+1=73$ satisfies both conditions. We subtract the biggest multiple of 11 less than 73 to get the remainder. Thus, $73-11(6)=73-66=\boxed{\text{(E) }10}$.

2016 AMC 8 (ProblemsAnswer KeyResources)
Preceded by
Problem 4
Followed by
Problem 6
1 2 3 4 5 6 7 8 9 10 11 12 13 14 15 16 17 18 19 20 21 22 23 24 25
All AJHSME/AMC 8 Problems and Solutions

The problems on this page are copyrighted by the Mathematical Association of America's American Mathematics Competitions. AMC logo.png